Proving a second order special limit without derivatives












0












$begingroup$


The special limit
$$
lim_{x to 0} frac{e^x-x-1}{x^2}=frac 1 2
$$

can be proved by Taylor expansion or with L'Hôpital's rule. Is it possoble to prove it without using derivatives?










share|cite|improve this question











$endgroup$








  • 2




    $begingroup$
    Is there a reason that neither of these methods can be used?
    $endgroup$
    – Henry Lee
    Dec 2 '18 at 18:54






  • 1




    $begingroup$
    If we define $e^x$ as $sum_{ngeq 0}frac{x^n}{n!}$ that is trivial.
    $endgroup$
    – Jack D'Aurizio
    Dec 2 '18 at 19:07










  • $begingroup$
    See math.stackexchange.com/questions/387333/…
    $endgroup$
    – lab bhattacharjee
    Dec 5 '18 at 8:45
















0












$begingroup$


The special limit
$$
lim_{x to 0} frac{e^x-x-1}{x^2}=frac 1 2
$$

can be proved by Taylor expansion or with L'Hôpital's rule. Is it possoble to prove it without using derivatives?










share|cite|improve this question











$endgroup$








  • 2




    $begingroup$
    Is there a reason that neither of these methods can be used?
    $endgroup$
    – Henry Lee
    Dec 2 '18 at 18:54






  • 1




    $begingroup$
    If we define $e^x$ as $sum_{ngeq 0}frac{x^n}{n!}$ that is trivial.
    $endgroup$
    – Jack D'Aurizio
    Dec 2 '18 at 19:07










  • $begingroup$
    See math.stackexchange.com/questions/387333/…
    $endgroup$
    – lab bhattacharjee
    Dec 5 '18 at 8:45














0












0








0





$begingroup$


The special limit
$$
lim_{x to 0} frac{e^x-x-1}{x^2}=frac 1 2
$$

can be proved by Taylor expansion or with L'Hôpital's rule. Is it possoble to prove it without using derivatives?










share|cite|improve this question











$endgroup$




The special limit
$$
lim_{x to 0} frac{e^x-x-1}{x^2}=frac 1 2
$$

can be proved by Taylor expansion or with L'Hôpital's rule. Is it possoble to prove it without using derivatives?







real-analysis limits limits-without-lhopital






share|cite|improve this question















share|cite|improve this question













share|cite|improve this question




share|cite|improve this question








edited Dec 2 '18 at 19:38









gimusi

92.9k94494




92.9k94494










asked Dec 2 '18 at 18:52









Marco DisceMarco Disce

1,3961216




1,3961216








  • 2




    $begingroup$
    Is there a reason that neither of these methods can be used?
    $endgroup$
    – Henry Lee
    Dec 2 '18 at 18:54






  • 1




    $begingroup$
    If we define $e^x$ as $sum_{ngeq 0}frac{x^n}{n!}$ that is trivial.
    $endgroup$
    – Jack D'Aurizio
    Dec 2 '18 at 19:07










  • $begingroup$
    See math.stackexchange.com/questions/387333/…
    $endgroup$
    – lab bhattacharjee
    Dec 5 '18 at 8:45














  • 2




    $begingroup$
    Is there a reason that neither of these methods can be used?
    $endgroup$
    – Henry Lee
    Dec 2 '18 at 18:54






  • 1




    $begingroup$
    If we define $e^x$ as $sum_{ngeq 0}frac{x^n}{n!}$ that is trivial.
    $endgroup$
    – Jack D'Aurizio
    Dec 2 '18 at 19:07










  • $begingroup$
    See math.stackexchange.com/questions/387333/…
    $endgroup$
    – lab bhattacharjee
    Dec 5 '18 at 8:45








2




2




$begingroup$
Is there a reason that neither of these methods can be used?
$endgroup$
– Henry Lee
Dec 2 '18 at 18:54




$begingroup$
Is there a reason that neither of these methods can be used?
$endgroup$
– Henry Lee
Dec 2 '18 at 18:54




1




1




$begingroup$
If we define $e^x$ as $sum_{ngeq 0}frac{x^n}{n!}$ that is trivial.
$endgroup$
– Jack D'Aurizio
Dec 2 '18 at 19:07




$begingroup$
If we define $e^x$ as $sum_{ngeq 0}frac{x^n}{n!}$ that is trivial.
$endgroup$
– Jack D'Aurizio
Dec 2 '18 at 19:07












$begingroup$
See math.stackexchange.com/questions/387333/…
$endgroup$
– lab bhattacharjee
Dec 5 '18 at 8:45




$begingroup$
See math.stackexchange.com/questions/387333/…
$endgroup$
– lab bhattacharjee
Dec 5 '18 at 8:45










3 Answers
3






active

oldest

votes


















2












$begingroup$

$$frac{e^x-1-x}{x^2}=int_{0}^{1}(1-y)e^{xy},dy $$
hence by the dominated convergence theorem
$$ lim_{xto 0}frac{e^x-1-x}{x^2}=int_{0}^{1}(1-y),dy = frac{1}{2}.$$






share|cite|improve this answer









$endgroup$





















    1












    $begingroup$

    We have by $x=2y$



    $$frac{e^x-x-1}{x^2}=frac{e^{2y}-2y-1}{4y^2}=frac{(e^y-1)^2+2e^y-2y-2}{4y^2}=frac14left(frac{e^y-1}{y}right)^2+frac12frac{e^y-y-1}{y^2}$$



    therefore assuming that the limit exists we have



    $$L=frac14+frac12L implies L=frac12$$



    Refer also to




    • Are all limits solvable without L'Hôpital Rule or Series Expansion






    share|cite|improve this answer









    $endgroup$













    • $begingroup$
      Since L'Hôpital and Taylor expansion are explicitly ruled out, you might want to mention how $(e^y-1)/y to 1$ is obtained.
      $endgroup$
      – Martin R
      Dec 2 '18 at 21:25












    • $begingroup$
      @MartinR My interpretation is that we can't use l'Hopital and Taylor to solve the limit but I think we can use the well known standard limit which, as you can know I suppose, can be derived without Hopital and Taylor. I'll add a reference to that anyway. Thanks
      $endgroup$
      – gimusi
      Dec 2 '18 at 21:38



















    0












    $begingroup$

    Let $$f(x)=e^{sqrt{x}}-sqrt{x}$$



    Then your limit is



    $$lim_{Xto0^+}frac{f(X)-f(0)}{X-0}$$






    share|cite|improve this answer









    $endgroup$













    • $begingroup$
      In the limit expression, you mean $f(x^2)$.
      $endgroup$
      – Anurag A
      Dec 2 '18 at 19:01












    • $begingroup$
      @AnuragA No. {}{}{}{}{}{}.
      $endgroup$
      – hamam_Abdallah
      Dec 2 '18 at 19:01










    • $begingroup$
      is that by derivative?
      $endgroup$
      – gimusi
      Dec 2 '18 at 20:06











    Your Answer





    StackExchange.ifUsing("editor", function () {
    return StackExchange.using("mathjaxEditing", function () {
    StackExchange.MarkdownEditor.creationCallbacks.add(function (editor, postfix) {
    StackExchange.mathjaxEditing.prepareWmdForMathJax(editor, postfix, [["$", "$"], ["\\(","\\)"]]);
    });
    });
    }, "mathjax-editing");

    StackExchange.ready(function() {
    var channelOptions = {
    tags: "".split(" "),
    id: "69"
    };
    initTagRenderer("".split(" "), "".split(" "), channelOptions);

    StackExchange.using("externalEditor", function() {
    // Have to fire editor after snippets, if snippets enabled
    if (StackExchange.settings.snippets.snippetsEnabled) {
    StackExchange.using("snippets", function() {
    createEditor();
    });
    }
    else {
    createEditor();
    }
    });

    function createEditor() {
    StackExchange.prepareEditor({
    heartbeatType: 'answer',
    autoActivateHeartbeat: false,
    convertImagesToLinks: true,
    noModals: true,
    showLowRepImageUploadWarning: true,
    reputationToPostImages: 10,
    bindNavPrevention: true,
    postfix: "",
    imageUploader: {
    brandingHtml: "Powered by u003ca class="icon-imgur-white" href="https://imgur.com/"u003eu003c/au003e",
    contentPolicyHtml: "User contributions licensed under u003ca href="https://creativecommons.org/licenses/by-sa/3.0/"u003ecc by-sa 3.0 with attribution requiredu003c/au003e u003ca href="https://stackoverflow.com/legal/content-policy"u003e(content policy)u003c/au003e",
    allowUrls: true
    },
    noCode: true, onDemand: true,
    discardSelector: ".discard-answer"
    ,immediatelyShowMarkdownHelp:true
    });


    }
    });














    draft saved

    draft discarded


















    StackExchange.ready(
    function () {
    StackExchange.openid.initPostLogin('.new-post-login', 'https%3a%2f%2fmath.stackexchange.com%2fquestions%2f3023041%2fproving-a-second-order-special-limit-without-derivatives%23new-answer', 'question_page');
    }
    );

    Post as a guest















    Required, but never shown

























    3 Answers
    3






    active

    oldest

    votes








    3 Answers
    3






    active

    oldest

    votes









    active

    oldest

    votes






    active

    oldest

    votes









    2












    $begingroup$

    $$frac{e^x-1-x}{x^2}=int_{0}^{1}(1-y)e^{xy},dy $$
    hence by the dominated convergence theorem
    $$ lim_{xto 0}frac{e^x-1-x}{x^2}=int_{0}^{1}(1-y),dy = frac{1}{2}.$$






    share|cite|improve this answer









    $endgroup$


















      2












      $begingroup$

      $$frac{e^x-1-x}{x^2}=int_{0}^{1}(1-y)e^{xy},dy $$
      hence by the dominated convergence theorem
      $$ lim_{xto 0}frac{e^x-1-x}{x^2}=int_{0}^{1}(1-y),dy = frac{1}{2}.$$






      share|cite|improve this answer









      $endgroup$
















        2












        2








        2





        $begingroup$

        $$frac{e^x-1-x}{x^2}=int_{0}^{1}(1-y)e^{xy},dy $$
        hence by the dominated convergence theorem
        $$ lim_{xto 0}frac{e^x-1-x}{x^2}=int_{0}^{1}(1-y),dy = frac{1}{2}.$$






        share|cite|improve this answer









        $endgroup$



        $$frac{e^x-1-x}{x^2}=int_{0}^{1}(1-y)e^{xy},dy $$
        hence by the dominated convergence theorem
        $$ lim_{xto 0}frac{e^x-1-x}{x^2}=int_{0}^{1}(1-y),dy = frac{1}{2}.$$







        share|cite|improve this answer












        share|cite|improve this answer



        share|cite|improve this answer










        answered Dec 2 '18 at 19:10









        Jack D'AurizioJack D'Aurizio

        288k33280660




        288k33280660























            1












            $begingroup$

            We have by $x=2y$



            $$frac{e^x-x-1}{x^2}=frac{e^{2y}-2y-1}{4y^2}=frac{(e^y-1)^2+2e^y-2y-2}{4y^2}=frac14left(frac{e^y-1}{y}right)^2+frac12frac{e^y-y-1}{y^2}$$



            therefore assuming that the limit exists we have



            $$L=frac14+frac12L implies L=frac12$$



            Refer also to




            • Are all limits solvable without L'Hôpital Rule or Series Expansion






            share|cite|improve this answer









            $endgroup$













            • $begingroup$
              Since L'Hôpital and Taylor expansion are explicitly ruled out, you might want to mention how $(e^y-1)/y to 1$ is obtained.
              $endgroup$
              – Martin R
              Dec 2 '18 at 21:25












            • $begingroup$
              @MartinR My interpretation is that we can't use l'Hopital and Taylor to solve the limit but I think we can use the well known standard limit which, as you can know I suppose, can be derived without Hopital and Taylor. I'll add a reference to that anyway. Thanks
              $endgroup$
              – gimusi
              Dec 2 '18 at 21:38
















            1












            $begingroup$

            We have by $x=2y$



            $$frac{e^x-x-1}{x^2}=frac{e^{2y}-2y-1}{4y^2}=frac{(e^y-1)^2+2e^y-2y-2}{4y^2}=frac14left(frac{e^y-1}{y}right)^2+frac12frac{e^y-y-1}{y^2}$$



            therefore assuming that the limit exists we have



            $$L=frac14+frac12L implies L=frac12$$



            Refer also to




            • Are all limits solvable without L'Hôpital Rule or Series Expansion






            share|cite|improve this answer









            $endgroup$













            • $begingroup$
              Since L'Hôpital and Taylor expansion are explicitly ruled out, you might want to mention how $(e^y-1)/y to 1$ is obtained.
              $endgroup$
              – Martin R
              Dec 2 '18 at 21:25












            • $begingroup$
              @MartinR My interpretation is that we can't use l'Hopital and Taylor to solve the limit but I think we can use the well known standard limit which, as you can know I suppose, can be derived without Hopital and Taylor. I'll add a reference to that anyway. Thanks
              $endgroup$
              – gimusi
              Dec 2 '18 at 21:38














            1












            1








            1





            $begingroup$

            We have by $x=2y$



            $$frac{e^x-x-1}{x^2}=frac{e^{2y}-2y-1}{4y^2}=frac{(e^y-1)^2+2e^y-2y-2}{4y^2}=frac14left(frac{e^y-1}{y}right)^2+frac12frac{e^y-y-1}{y^2}$$



            therefore assuming that the limit exists we have



            $$L=frac14+frac12L implies L=frac12$$



            Refer also to




            • Are all limits solvable without L'Hôpital Rule or Series Expansion






            share|cite|improve this answer









            $endgroup$



            We have by $x=2y$



            $$frac{e^x-x-1}{x^2}=frac{e^{2y}-2y-1}{4y^2}=frac{(e^y-1)^2+2e^y-2y-2}{4y^2}=frac14left(frac{e^y-1}{y}right)^2+frac12frac{e^y-y-1}{y^2}$$



            therefore assuming that the limit exists we have



            $$L=frac14+frac12L implies L=frac12$$



            Refer also to




            • Are all limits solvable without L'Hôpital Rule or Series Expansion







            share|cite|improve this answer












            share|cite|improve this answer



            share|cite|improve this answer










            answered Dec 2 '18 at 19:11









            gimusigimusi

            92.9k94494




            92.9k94494












            • $begingroup$
              Since L'Hôpital and Taylor expansion are explicitly ruled out, you might want to mention how $(e^y-1)/y to 1$ is obtained.
              $endgroup$
              – Martin R
              Dec 2 '18 at 21:25












            • $begingroup$
              @MartinR My interpretation is that we can't use l'Hopital and Taylor to solve the limit but I think we can use the well known standard limit which, as you can know I suppose, can be derived without Hopital and Taylor. I'll add a reference to that anyway. Thanks
              $endgroup$
              – gimusi
              Dec 2 '18 at 21:38


















            • $begingroup$
              Since L'Hôpital and Taylor expansion are explicitly ruled out, you might want to mention how $(e^y-1)/y to 1$ is obtained.
              $endgroup$
              – Martin R
              Dec 2 '18 at 21:25












            • $begingroup$
              @MartinR My interpretation is that we can't use l'Hopital and Taylor to solve the limit but I think we can use the well known standard limit which, as you can know I suppose, can be derived without Hopital and Taylor. I'll add a reference to that anyway. Thanks
              $endgroup$
              – gimusi
              Dec 2 '18 at 21:38
















            $begingroup$
            Since L'Hôpital and Taylor expansion are explicitly ruled out, you might want to mention how $(e^y-1)/y to 1$ is obtained.
            $endgroup$
            – Martin R
            Dec 2 '18 at 21:25






            $begingroup$
            Since L'Hôpital and Taylor expansion are explicitly ruled out, you might want to mention how $(e^y-1)/y to 1$ is obtained.
            $endgroup$
            – Martin R
            Dec 2 '18 at 21:25














            $begingroup$
            @MartinR My interpretation is that we can't use l'Hopital and Taylor to solve the limit but I think we can use the well known standard limit which, as you can know I suppose, can be derived without Hopital and Taylor. I'll add a reference to that anyway. Thanks
            $endgroup$
            – gimusi
            Dec 2 '18 at 21:38




            $begingroup$
            @MartinR My interpretation is that we can't use l'Hopital and Taylor to solve the limit but I think we can use the well known standard limit which, as you can know I suppose, can be derived without Hopital and Taylor. I'll add a reference to that anyway. Thanks
            $endgroup$
            – gimusi
            Dec 2 '18 at 21:38











            0












            $begingroup$

            Let $$f(x)=e^{sqrt{x}}-sqrt{x}$$



            Then your limit is



            $$lim_{Xto0^+}frac{f(X)-f(0)}{X-0}$$






            share|cite|improve this answer









            $endgroup$













            • $begingroup$
              In the limit expression, you mean $f(x^2)$.
              $endgroup$
              – Anurag A
              Dec 2 '18 at 19:01












            • $begingroup$
              @AnuragA No. {}{}{}{}{}{}.
              $endgroup$
              – hamam_Abdallah
              Dec 2 '18 at 19:01










            • $begingroup$
              is that by derivative?
              $endgroup$
              – gimusi
              Dec 2 '18 at 20:06
















            0












            $begingroup$

            Let $$f(x)=e^{sqrt{x}}-sqrt{x}$$



            Then your limit is



            $$lim_{Xto0^+}frac{f(X)-f(0)}{X-0}$$






            share|cite|improve this answer









            $endgroup$













            • $begingroup$
              In the limit expression, you mean $f(x^2)$.
              $endgroup$
              – Anurag A
              Dec 2 '18 at 19:01












            • $begingroup$
              @AnuragA No. {}{}{}{}{}{}.
              $endgroup$
              – hamam_Abdallah
              Dec 2 '18 at 19:01










            • $begingroup$
              is that by derivative?
              $endgroup$
              – gimusi
              Dec 2 '18 at 20:06














            0












            0








            0





            $begingroup$

            Let $$f(x)=e^{sqrt{x}}-sqrt{x}$$



            Then your limit is



            $$lim_{Xto0^+}frac{f(X)-f(0)}{X-0}$$






            share|cite|improve this answer









            $endgroup$



            Let $$f(x)=e^{sqrt{x}}-sqrt{x}$$



            Then your limit is



            $$lim_{Xto0^+}frac{f(X)-f(0)}{X-0}$$







            share|cite|improve this answer












            share|cite|improve this answer



            share|cite|improve this answer










            answered Dec 2 '18 at 18:59









            hamam_Abdallahhamam_Abdallah

            38k21634




            38k21634












            • $begingroup$
              In the limit expression, you mean $f(x^2)$.
              $endgroup$
              – Anurag A
              Dec 2 '18 at 19:01












            • $begingroup$
              @AnuragA No. {}{}{}{}{}{}.
              $endgroup$
              – hamam_Abdallah
              Dec 2 '18 at 19:01










            • $begingroup$
              is that by derivative?
              $endgroup$
              – gimusi
              Dec 2 '18 at 20:06


















            • $begingroup$
              In the limit expression, you mean $f(x^2)$.
              $endgroup$
              – Anurag A
              Dec 2 '18 at 19:01












            • $begingroup$
              @AnuragA No. {}{}{}{}{}{}.
              $endgroup$
              – hamam_Abdallah
              Dec 2 '18 at 19:01










            • $begingroup$
              is that by derivative?
              $endgroup$
              – gimusi
              Dec 2 '18 at 20:06
















            $begingroup$
            In the limit expression, you mean $f(x^2)$.
            $endgroup$
            – Anurag A
            Dec 2 '18 at 19:01






            $begingroup$
            In the limit expression, you mean $f(x^2)$.
            $endgroup$
            – Anurag A
            Dec 2 '18 at 19:01














            $begingroup$
            @AnuragA No. {}{}{}{}{}{}.
            $endgroup$
            – hamam_Abdallah
            Dec 2 '18 at 19:01




            $begingroup$
            @AnuragA No. {}{}{}{}{}{}.
            $endgroup$
            – hamam_Abdallah
            Dec 2 '18 at 19:01












            $begingroup$
            is that by derivative?
            $endgroup$
            – gimusi
            Dec 2 '18 at 20:06




            $begingroup$
            is that by derivative?
            $endgroup$
            – gimusi
            Dec 2 '18 at 20:06


















            draft saved

            draft discarded




















































            Thanks for contributing an answer to Mathematics Stack Exchange!


            • Please be sure to answer the question. Provide details and share your research!

            But avoid



            • Asking for help, clarification, or responding to other answers.

            • Making statements based on opinion; back them up with references or personal experience.


            Use MathJax to format equations. MathJax reference.


            To learn more, see our tips on writing great answers.




            draft saved


            draft discarded














            StackExchange.ready(
            function () {
            StackExchange.openid.initPostLogin('.new-post-login', 'https%3a%2f%2fmath.stackexchange.com%2fquestions%2f3023041%2fproving-a-second-order-special-limit-without-derivatives%23new-answer', 'question_page');
            }
            );

            Post as a guest















            Required, but never shown





















































            Required, but never shown














            Required, but never shown












            Required, but never shown







            Required, but never shown

































            Required, but never shown














            Required, but never shown












            Required, but never shown







            Required, but never shown







            Popular posts from this blog

            Le Mesnil-Réaume

            Ida-Boy-Ed-Garten

            web3.py web3.isConnected() returns false always